LSAT and Law School Admissions Forum

Get expert LSAT preparation and law school admissions advice from PowerScore Test Preparation.

 Administrator
PowerScore Staff
  • PowerScore Staff
  • Posts: 8950
  • Joined: Feb 02, 2011
|
#24681
Complete Question Explanation

Must Be True. The correct answer choice is (A)

This forester tells the story of the world’s remaining forests, which have significantly dwindled from what they once were. They are home to many endangered species but are not self sustaining. The final sentence in the stimulus is a conditional statement:

To maintain all of their respective plant and animal species, these forests require resource manager intervention:
  • maintain all species ..... :arrow: ..... resource manager intervention
The contrapositive: ..... intervention ..... :arrow: ..... can maintain all species

Only correct answer choice (A) is confirmed by the information presented in the stimulus. This is the correct answer choice, restating the contrapositive diagrammed above. If there is no intervention, not every species can be maintained.
 ylikate
  • Posts: 30
  • Joined: Aug 27, 2013
|
#11636
Would B have been correct if it says"Some (instead of "Many") of the world's most engangered species will not survive?
Or are there some other problems with choice B?

Thanks
 Steve Stein
PowerScore Staff
  • PowerScore Staff
  • Posts: 1153
  • Joined: Apr 11, 2011
|
#11640
Hi Kate,

Good question. "Some" and "many" are similarly vague--that is not the issue with (B)

The author has provided that the majority of forests are sickly fragments, yet they include the last of some endangered species. That would allow for the possibility that two or three forests in the world, for example, have endangered species.

Choice (B) goes too far, saying that without regular intervention in most of the forests in the world, endangered species will die. We might just have to intervene in two or three forests in order to save those species referenced by Forester.

I hope that's helpful! Please let me know whether this is clear--thanks!

~Steve
 ylikate
  • Posts: 30
  • Joined: Aug 27, 2013
|
#11646
so of the majority of forests that are fragmented, maybe only 1 or 2 supports endangered species. Therefore it's not necessary, as B states, to intervene majority of the forest. Am I understanding it correctly?
 Steve Stein
PowerScore Staff
  • PowerScore Staff
  • Posts: 1153
  • Joined: Apr 11, 2011
|
#11651
Hey Kate,

That's a tricky wrong answer choice, but you got it!

~Steve
 bk1111
  • Posts: 103
  • Joined: Apr 22, 2017
|
#34926
Steve Stein wrote:Hi Kate,

Good question. "Some" and "many" are similarly vague--that is not the issue with (B)

The author has provided that the majority of forests are sickly fragments, yet they include the last of some endangered species. That would allow for the possibility that two or three forests in the world, for example, have endangered species.

Choice (B) goes too far, saying that without regular intervention in most of the forests in the world, endangered species will die. We might just have to intervene in two or three forests in order to save those species referenced by Forester.

I hope that's helpful! Please let me know whether this is clear--thanks!

~Steve
Hi, I am confused about this. "A" also states 53% "most of the world's forests" will lose some animal species and you pointed out B is incorrect because of "most of the forests in the world". Isn't that the same thing?

I did not choose A because it said "if no one intervenes"...I thought that was too general. Why does that not matter? Thanks
 Charlie Melman
PowerScore Staff
  • PowerScore Staff
  • Posts: 85
  • Joined: Feb 10, 2017
|
#35149
Hi bk1111,

This is a very tough question because answer choices (A), and (B) are very similar.

(B) translates to "Not intervene :arrow: Many of the world's most endangered species will not survive.

The problem here is that the stimulus talks about the "full complement" of species, while this answer choice talks about "endangered species." Maybe the endangered species will survive even though the full complement won't.

Hope this helps!
 lilmissunshine
  • Posts: 94
  • Joined: Jun 07, 2018
|
#46911
Charlie Melman wrote:The problem here is that the stimulus talks about the "full complement" of species, while this answer choice talks about "endangered species." Maybe the endangered species will survive even though the full complement won't.
Hi Charlie,

Are you saying maybe some species that are not the most endangered won't survive? In other words, the world's most endangered species are not necessarily the ones that won't survive. So is the flaw in answer choice (B) not "most of the world's remaining forests"? To be honest I found the previous explanation a bit confusing...

Many thanks!
 Alex Bodaken
PowerScore Staff
  • PowerScore Staff
  • Posts: 135
  • Joined: Feb 21, 2018
|
#47043
lilmissunshine,

Thanks for the question! You're on the right track re: Charlie's explanation: the issue is that while the stimulus says that without resource manager intervention, a "full complement" of plants and animals will not survive, answer choice (B) says specifically that it will be the endangered species that will not survive without intervention. We simply can't make that leap: as you note, perhaps it is the non-endangered species that will not survive rather than the endangered species...we don't know. That lack of provability is what makes (B) an incorrect answer choice.

Hope that helps!
Alex
 Katherinthesky
  • Posts: 36
  • Joined: Feb 07, 2020
|
#85927
Is (D) incorrect because of the "always," which cannot be supported?

Thanks in advance.

Get the most out of your LSAT Prep Plus subscription.

Analyze and track your performance with our Testing and Analytics Package.